Quick question on repeated roots when solving differential equations

Click For Summary
The discussion centers on identifying repeated roots in polynomial equations, specifically in the context of solving differential equations. The example provided shows the factorization of the polynomial, leading to roots of 2, 2, 3, and -3, confirming that 2 is a repeated root. A subsequent polynomial, (λ² - 1)(λ + 1), is analyzed to determine if -1 is a repeated root. It is clarified that -1 is indeed a double root, leading to the conclusion that the values for λ are 1, -1, -1. Understanding repeated roots is crucial for correctly formulating solutions in differential equations.
rwooduk
Messages
757
Reaction score
59
say we have gone through the steps and have...

##(\lambda - 2)^{2}(\lambda ^{2}-9) = 0##

which we can write as...

##(\lambda - 2)(\lambda - 2)(\lambda ^{2}-9) = 0##

we have value for lambda of 2, 2, 3, -3

because we have a repeated root.

now, say we have

##(\lambda^{2} - 1)(\lambda + 1) = 0##

my question is would you have 2 values of -1 like the example above i.e. 1, -1, -1? or would you have values of lambda of just 1, -1? and if the former would you treat the -1 as a repeated root?

it becomes important when writing the solution because for a repeated root there an x in the second term with the exponential.

thanks in advance for any direction on this.
 
Physics news on Phys.org
This isn't really a question about differential equations, it is a question about basic algebra.

Since \lambda^2- 1= (\lambda- 1)(\lambda+ 1), which I am sure you already knew,
(\lambda^2- 1)(\lambda+ 1)= (\lambda- 1)(\lambda+ 1)(\lambda+ 1)= (\lambda- 1)(\lambda+ 1)^2
so, yes, -1 is a double root.
 
  • Like
Likes 1 person
ahh, never thought of it that way! that's great, many thanks!
 
Question: A clock's minute hand has length 4 and its hour hand has length 3. What is the distance between the tips at the moment when it is increasing most rapidly?(Putnam Exam Question) Answer: Making assumption that both the hands moves at constant angular velocities, the answer is ## \sqrt{7} .## But don't you think this assumption is somewhat doubtful and wrong?

Similar threads

  • · Replies 6 ·
Replies
6
Views
1K
Replies
6
Views
2K
Replies
19
Views
1K
  • · Replies 7 ·
Replies
7
Views
1K
Replies
2
Views
2K
  • · Replies 4 ·
Replies
4
Views
2K
  • · Replies 2 ·
Replies
2
Views
2K
Replies
2
Views
2K
Replies
1
Views
1K
  • · Replies 9 ·
Replies
9
Views
2K